LSAT and Law School Admissions Forum

Get expert LSAT preparation and law school admissions advice from PowerScore Test Preparation.

 Administrator
PowerScore Staff
  • PowerScore Staff
  • Posts: 8916
  • Joined: Feb 02, 2011
|
#81058
Complete Question Explanation

Strengthen. The correct answer choice is (B).

Answer choice (A):

Answer choice (B): This it the correct answer choice.

Answer choice (C):

Answer choice (D):

Answer choice (E):

This explanation is still in progress. Please post any questions below!
 maximbasu
  • Posts: 59
  • Joined: May 19, 2016
|
#28166
Hi,

I chose answer choice E while the correct ANS is B.

The stimulus states:
1. The view that people didn't see my film because it got 2 bad reviews is wrong!
2. Why? The audience sucked: it was small + they went to a variety of films similar to mine
2. My film is amazing + society is to blame

Task: Support

My reasoning for E: If people love watching a "variety" of films, then that would directly support the filmmaker. His film got a small # of people because they didn't all want to watch his film; it is a Law of Nature.
Is E wrong because by "kinds" of film the LSAT writers meant "types of film?" Then, it would weaken the conclusion, because the "same type of filmgoer" would want variety in genre.

Is B correct because it directly talks about the timeframe: "the weekend." Since all of the people, who watch the same type of movie, went to see one film, obviously he won't get all of them to see his movie. That's impossible.

MB
User avatar
 Jonathan Evans
PowerScore Staff
  • PowerScore Staff
  • Posts: 726
  • Joined: Jun 09, 2016
|
#28407
Hi, Max,

It's possible that you are "overthinking" this stimulus and perhaps getting lost in unnecessary digressions. When you work a stimulus such as this, try to distill it down to its key components. You've done a good job breaking down this argument, but let me add a couple suggestions:

Start with the conclusion:
Filmgoers did not stay away from the movie because of the negative reviews.
Consider the support the film director offers:
The film opened the same weekend as did several other films that share the same audience.

The number of such viewers is small.
Now notice any possible flaws or gaps in the reasoning. The director assumes without providing justification that the limited audience size is zero sum, that more films mean a divided audience. He makes a causal argument.

Since the task is to support the director's conclusion, prephrase first by recognizing the type of information that you're looking for, second by thinking of any obvious ways to accomplish this task.

For example, here you could prephrase that you want something that will really show that it was the other movies' openings that were to blame for the poor performance and not the negative reviews. Since the argument involves a causal fallacy, you could brainstorm that you might like evidence that suggests that any time multiple such movies are released on the same weekend, performance drops for all films relative to performance of similar films on other, less crowded weekends. Etc. Etc.

In this case, answer choice B does provide support to suggest that filmgoers are not inclined to see multiple movies on the same weekend, bolstering the case that the problem was the movies rather than the reviews.

Answer choice E does not directly address the gap here or provide reasons why the other films were the problem and not the negative reviews. This answer is actually out of scope. To avoid other similar situations, try to limit assumptions you bring into LR problems. Try to recognize when you have to introduce an intermediate step between any given answer choice and the task you wish to accomplish. Further, rely on strict clarity about an argument's components and develop strong prephrasing skills to avoid falling into traps.

Please let me know how I may be of further help.

Get the most out of your LSAT Prep Plus subscription.

Analyze and track your performance with our Testing and Analytics Package.